Confused about open and closed set in $mathbb{R}^2$












0












$begingroup$


While this question is basically meant for all $mathbb{R}^n$ where $r > 1$, I suppose it would be the easiest to grasp the concept in $mathbb{R}^2$ and conclude the analog to $mathbb{R}^2$.



So I was wondering since we just started out with Topology, how to find out if a set is closed or open in $mathbb{R}^2$.



Let's say I have a set $A subset mathbb{R}$ defined as:



$$A=(0,1) times [0,1]$$



Since I now have a set "composed" by an open and a closed set I wonder if $A$ is now called open or closed.



Or is this a case where it neither is open nor closed?










share|cite|improve this question











$endgroup$








  • 1




    $begingroup$
    It is neither open nor closed. This is true of most subsets in a sense.
    $endgroup$
    – Matt Samuel
    Dec 15 '18 at 12:53
















0












$begingroup$


While this question is basically meant for all $mathbb{R}^n$ where $r > 1$, I suppose it would be the easiest to grasp the concept in $mathbb{R}^2$ and conclude the analog to $mathbb{R}^2$.



So I was wondering since we just started out with Topology, how to find out if a set is closed or open in $mathbb{R}^2$.



Let's say I have a set $A subset mathbb{R}$ defined as:



$$A=(0,1) times [0,1]$$



Since I now have a set "composed" by an open and a closed set I wonder if $A$ is now called open or closed.



Or is this a case where it neither is open nor closed?










share|cite|improve this question











$endgroup$








  • 1




    $begingroup$
    It is neither open nor closed. This is true of most subsets in a sense.
    $endgroup$
    – Matt Samuel
    Dec 15 '18 at 12:53














0












0








0





$begingroup$


While this question is basically meant for all $mathbb{R}^n$ where $r > 1$, I suppose it would be the easiest to grasp the concept in $mathbb{R}^2$ and conclude the analog to $mathbb{R}^2$.



So I was wondering since we just started out with Topology, how to find out if a set is closed or open in $mathbb{R}^2$.



Let's say I have a set $A subset mathbb{R}$ defined as:



$$A=(0,1) times [0,1]$$



Since I now have a set "composed" by an open and a closed set I wonder if $A$ is now called open or closed.



Or is this a case where it neither is open nor closed?










share|cite|improve this question











$endgroup$




While this question is basically meant for all $mathbb{R}^n$ where $r > 1$, I suppose it would be the easiest to grasp the concept in $mathbb{R}^2$ and conclude the analog to $mathbb{R}^2$.



So I was wondering since we just started out with Topology, how to find out if a set is closed or open in $mathbb{R}^2$.



Let's say I have a set $A subset mathbb{R}$ defined as:



$$A=(0,1) times [0,1]$$



Since I now have a set "composed" by an open and a closed set I wonder if $A$ is now called open or closed.



Or is this a case where it neither is open nor closed?







general-topology






share|cite|improve this question















share|cite|improve this question













share|cite|improve this question




share|cite|improve this question








edited Dec 15 '18 at 13:08









Andrés E. Caicedo

65.4k8158249




65.4k8158249










asked Dec 15 '18 at 12:51









MLKMLK

80112




80112








  • 1




    $begingroup$
    It is neither open nor closed. This is true of most subsets in a sense.
    $endgroup$
    – Matt Samuel
    Dec 15 '18 at 12:53














  • 1




    $begingroup$
    It is neither open nor closed. This is true of most subsets in a sense.
    $endgroup$
    – Matt Samuel
    Dec 15 '18 at 12:53








1




1




$begingroup$
It is neither open nor closed. This is true of most subsets in a sense.
$endgroup$
– Matt Samuel
Dec 15 '18 at 12:53




$begingroup$
It is neither open nor closed. This is true of most subsets in a sense.
$endgroup$
– Matt Samuel
Dec 15 '18 at 12:53










2 Answers
2






active

oldest

votes


















1












$begingroup$


  • Since $(0,0)notin A$ but every open ball centered at $(0,0)$ contains points of $A$, $A$ is not closed.

  • Since $left(frac12,0right)in A$ but every open ball centered at $left(frac12,0right)in A$ contains points out of $A$, $A$ is not open.


Therefore, $A$ is neither closed nor open.






share|cite|improve this answer









$endgroup$













  • $begingroup$
    Interesting. Thank you. So basically for every $mathbb{R}^n$ one just has to look at every set that is making up the set $A$ and if one is open while one is closed it is neither the one nor the other
    $endgroup$
    – MLK
    Dec 15 '18 at 12:58








  • 1




    $begingroup$
    Yes, but that's an almost useless piece of information, since very few subsets of $mathbb{R}^2$ can be expressed as the product of two subsets of $mathbb R$.
    $endgroup$
    – José Carlos Santos
    Dec 15 '18 at 13:45



















1












$begingroup$

This set $(0,1) times [0,1]$ is not closed and not open.



Not closed e.g. because $(frac{1}{n}, frac{1}{n})$ is a sequence in it conervging to $(0,0) notin A$. Not open e.g. because $(frac{1}{2}, -frac{1}{n})$ is a sequence converging to $(frac12,0)$ in $A$ but all of whose points lie outside $A$.



Another way: projections are open maps so if $A$ were open, so would $pi_2[A]= [0,1]$ be in $mathbb{R}$, which it is not. If $A$ were closed, it would be compact (closed and bounded in $mathbb{R}^2$) but $pi_1[A] = (0,1)$ is not compact (not even closed).






share|cite|improve this answer









$endgroup$













    Your Answer





    StackExchange.ifUsing("editor", function () {
    return StackExchange.using("mathjaxEditing", function () {
    StackExchange.MarkdownEditor.creationCallbacks.add(function (editor, postfix) {
    StackExchange.mathjaxEditing.prepareWmdForMathJax(editor, postfix, [["$", "$"], ["\\(","\\)"]]);
    });
    });
    }, "mathjax-editing");

    StackExchange.ready(function() {
    var channelOptions = {
    tags: "".split(" "),
    id: "69"
    };
    initTagRenderer("".split(" "), "".split(" "), channelOptions);

    StackExchange.using("externalEditor", function() {
    // Have to fire editor after snippets, if snippets enabled
    if (StackExchange.settings.snippets.snippetsEnabled) {
    StackExchange.using("snippets", function() {
    createEditor();
    });
    }
    else {
    createEditor();
    }
    });

    function createEditor() {
    StackExchange.prepareEditor({
    heartbeatType: 'answer',
    autoActivateHeartbeat: false,
    convertImagesToLinks: true,
    noModals: true,
    showLowRepImageUploadWarning: true,
    reputationToPostImages: 10,
    bindNavPrevention: true,
    postfix: "",
    imageUploader: {
    brandingHtml: "Powered by u003ca class="icon-imgur-white" href="https://imgur.com/"u003eu003c/au003e",
    contentPolicyHtml: "User contributions licensed under u003ca href="https://creativecommons.org/licenses/by-sa/3.0/"u003ecc by-sa 3.0 with attribution requiredu003c/au003e u003ca href="https://stackoverflow.com/legal/content-policy"u003e(content policy)u003c/au003e",
    allowUrls: true
    },
    noCode: true, onDemand: true,
    discardSelector: ".discard-answer"
    ,immediatelyShowMarkdownHelp:true
    });


    }
    });














    draft saved

    draft discarded


















    StackExchange.ready(
    function () {
    StackExchange.openid.initPostLogin('.new-post-login', 'https%3a%2f%2fmath.stackexchange.com%2fquestions%2f3040479%2fconfused-about-open-and-closed-set-in-mathbbr2%23new-answer', 'question_page');
    }
    );

    Post as a guest















    Required, but never shown

























    2 Answers
    2






    active

    oldest

    votes








    2 Answers
    2






    active

    oldest

    votes









    active

    oldest

    votes






    active

    oldest

    votes









    1












    $begingroup$


    • Since $(0,0)notin A$ but every open ball centered at $(0,0)$ contains points of $A$, $A$ is not closed.

    • Since $left(frac12,0right)in A$ but every open ball centered at $left(frac12,0right)in A$ contains points out of $A$, $A$ is not open.


    Therefore, $A$ is neither closed nor open.






    share|cite|improve this answer









    $endgroup$













    • $begingroup$
      Interesting. Thank you. So basically for every $mathbb{R}^n$ one just has to look at every set that is making up the set $A$ and if one is open while one is closed it is neither the one nor the other
      $endgroup$
      – MLK
      Dec 15 '18 at 12:58








    • 1




      $begingroup$
      Yes, but that's an almost useless piece of information, since very few subsets of $mathbb{R}^2$ can be expressed as the product of two subsets of $mathbb R$.
      $endgroup$
      – José Carlos Santos
      Dec 15 '18 at 13:45
















    1












    $begingroup$


    • Since $(0,0)notin A$ but every open ball centered at $(0,0)$ contains points of $A$, $A$ is not closed.

    • Since $left(frac12,0right)in A$ but every open ball centered at $left(frac12,0right)in A$ contains points out of $A$, $A$ is not open.


    Therefore, $A$ is neither closed nor open.






    share|cite|improve this answer









    $endgroup$













    • $begingroup$
      Interesting. Thank you. So basically for every $mathbb{R}^n$ one just has to look at every set that is making up the set $A$ and if one is open while one is closed it is neither the one nor the other
      $endgroup$
      – MLK
      Dec 15 '18 at 12:58








    • 1




      $begingroup$
      Yes, but that's an almost useless piece of information, since very few subsets of $mathbb{R}^2$ can be expressed as the product of two subsets of $mathbb R$.
      $endgroup$
      – José Carlos Santos
      Dec 15 '18 at 13:45














    1












    1








    1





    $begingroup$


    • Since $(0,0)notin A$ but every open ball centered at $(0,0)$ contains points of $A$, $A$ is not closed.

    • Since $left(frac12,0right)in A$ but every open ball centered at $left(frac12,0right)in A$ contains points out of $A$, $A$ is not open.


    Therefore, $A$ is neither closed nor open.






    share|cite|improve this answer









    $endgroup$




    • Since $(0,0)notin A$ but every open ball centered at $(0,0)$ contains points of $A$, $A$ is not closed.

    • Since $left(frac12,0right)in A$ but every open ball centered at $left(frac12,0right)in A$ contains points out of $A$, $A$ is not open.


    Therefore, $A$ is neither closed nor open.







    share|cite|improve this answer












    share|cite|improve this answer



    share|cite|improve this answer










    answered Dec 15 '18 at 12:55









    José Carlos SantosJosé Carlos Santos

    160k22127232




    160k22127232












    • $begingroup$
      Interesting. Thank you. So basically for every $mathbb{R}^n$ one just has to look at every set that is making up the set $A$ and if one is open while one is closed it is neither the one nor the other
      $endgroup$
      – MLK
      Dec 15 '18 at 12:58








    • 1




      $begingroup$
      Yes, but that's an almost useless piece of information, since very few subsets of $mathbb{R}^2$ can be expressed as the product of two subsets of $mathbb R$.
      $endgroup$
      – José Carlos Santos
      Dec 15 '18 at 13:45


















    • $begingroup$
      Interesting. Thank you. So basically for every $mathbb{R}^n$ one just has to look at every set that is making up the set $A$ and if one is open while one is closed it is neither the one nor the other
      $endgroup$
      – MLK
      Dec 15 '18 at 12:58








    • 1




      $begingroup$
      Yes, but that's an almost useless piece of information, since very few subsets of $mathbb{R}^2$ can be expressed as the product of two subsets of $mathbb R$.
      $endgroup$
      – José Carlos Santos
      Dec 15 '18 at 13:45
















    $begingroup$
    Interesting. Thank you. So basically for every $mathbb{R}^n$ one just has to look at every set that is making up the set $A$ and if one is open while one is closed it is neither the one nor the other
    $endgroup$
    – MLK
    Dec 15 '18 at 12:58






    $begingroup$
    Interesting. Thank you. So basically for every $mathbb{R}^n$ one just has to look at every set that is making up the set $A$ and if one is open while one is closed it is neither the one nor the other
    $endgroup$
    – MLK
    Dec 15 '18 at 12:58






    1




    1




    $begingroup$
    Yes, but that's an almost useless piece of information, since very few subsets of $mathbb{R}^2$ can be expressed as the product of two subsets of $mathbb R$.
    $endgroup$
    – José Carlos Santos
    Dec 15 '18 at 13:45




    $begingroup$
    Yes, but that's an almost useless piece of information, since very few subsets of $mathbb{R}^2$ can be expressed as the product of two subsets of $mathbb R$.
    $endgroup$
    – José Carlos Santos
    Dec 15 '18 at 13:45











    1












    $begingroup$

    This set $(0,1) times [0,1]$ is not closed and not open.



    Not closed e.g. because $(frac{1}{n}, frac{1}{n})$ is a sequence in it conervging to $(0,0) notin A$. Not open e.g. because $(frac{1}{2}, -frac{1}{n})$ is a sequence converging to $(frac12,0)$ in $A$ but all of whose points lie outside $A$.



    Another way: projections are open maps so if $A$ were open, so would $pi_2[A]= [0,1]$ be in $mathbb{R}$, which it is not. If $A$ were closed, it would be compact (closed and bounded in $mathbb{R}^2$) but $pi_1[A] = (0,1)$ is not compact (not even closed).






    share|cite|improve this answer









    $endgroup$


















      1












      $begingroup$

      This set $(0,1) times [0,1]$ is not closed and not open.



      Not closed e.g. because $(frac{1}{n}, frac{1}{n})$ is a sequence in it conervging to $(0,0) notin A$. Not open e.g. because $(frac{1}{2}, -frac{1}{n})$ is a sequence converging to $(frac12,0)$ in $A$ but all of whose points lie outside $A$.



      Another way: projections are open maps so if $A$ were open, so would $pi_2[A]= [0,1]$ be in $mathbb{R}$, which it is not. If $A$ were closed, it would be compact (closed and bounded in $mathbb{R}^2$) but $pi_1[A] = (0,1)$ is not compact (not even closed).






      share|cite|improve this answer









      $endgroup$
















        1












        1








        1





        $begingroup$

        This set $(0,1) times [0,1]$ is not closed and not open.



        Not closed e.g. because $(frac{1}{n}, frac{1}{n})$ is a sequence in it conervging to $(0,0) notin A$. Not open e.g. because $(frac{1}{2}, -frac{1}{n})$ is a sequence converging to $(frac12,0)$ in $A$ but all of whose points lie outside $A$.



        Another way: projections are open maps so if $A$ were open, so would $pi_2[A]= [0,1]$ be in $mathbb{R}$, which it is not. If $A$ were closed, it would be compact (closed and bounded in $mathbb{R}^2$) but $pi_1[A] = (0,1)$ is not compact (not even closed).






        share|cite|improve this answer









        $endgroup$



        This set $(0,1) times [0,1]$ is not closed and not open.



        Not closed e.g. because $(frac{1}{n}, frac{1}{n})$ is a sequence in it conervging to $(0,0) notin A$. Not open e.g. because $(frac{1}{2}, -frac{1}{n})$ is a sequence converging to $(frac12,0)$ in $A$ but all of whose points lie outside $A$.



        Another way: projections are open maps so if $A$ were open, so would $pi_2[A]= [0,1]$ be in $mathbb{R}$, which it is not. If $A$ were closed, it would be compact (closed and bounded in $mathbb{R}^2$) but $pi_1[A] = (0,1)$ is not compact (not even closed).







        share|cite|improve this answer












        share|cite|improve this answer



        share|cite|improve this answer










        answered Dec 15 '18 at 12:58









        Henno BrandsmaHenno Brandsma

        109k347114




        109k347114






























            draft saved

            draft discarded




















































            Thanks for contributing an answer to Mathematics Stack Exchange!


            • Please be sure to answer the question. Provide details and share your research!

            But avoid



            • Asking for help, clarification, or responding to other answers.

            • Making statements based on opinion; back them up with references or personal experience.


            Use MathJax to format equations. MathJax reference.


            To learn more, see our tips on writing great answers.




            draft saved


            draft discarded














            StackExchange.ready(
            function () {
            StackExchange.openid.initPostLogin('.new-post-login', 'https%3a%2f%2fmath.stackexchange.com%2fquestions%2f3040479%2fconfused-about-open-and-closed-set-in-mathbbr2%23new-answer', 'question_page');
            }
            );

            Post as a guest















            Required, but never shown





















































            Required, but never shown














            Required, but never shown












            Required, but never shown







            Required, but never shown

































            Required, but never shown














            Required, but never shown












            Required, but never shown







            Required, but never shown







            Popular posts from this blog

            Quarter-circle Tiles

            build a pushdown automaton that recognizes the reverse language of a given pushdown automaton?

            Mont Emei